Closure of a set in a metric space.












0












$begingroup$


I have a question with regards to the proof of the following:
Let $X = A cup A'$ be the closure of a set. $A'$ is the set of limit points of $A$. I wish to show that the closure is closed. I would very much like your opinions on the proof.



Let $ain A^ccap (A')^c$. Since $a$ is not a limit point of $A$ there exists a neighborhood of $a$ such that for every $q$ in the neighborhood of $a$, $q=a$ or $qnotin A$. This implies that $N_r(a)$ is contained within the complement of $A$. Now we would like to show that the neighborhood is contained within $(A')^c$, because then $N_r(a)$ $subset$ $A^c$ $cap$ $(A')^c$ which implies that the set is open, therefore the closure is closed. So assume that the neighborhood is not contained in $(A')^c$ this implies that $a$ $in$ $A'$ because $N_r(a)$ $subset$ $A'$, which is a contradiction.










share|cite|improve this question











$endgroup$












  • $begingroup$
    You start by basically assuming a not in X and proceed to make contradicition; thusly proclaiming X is the whole space, which usually it isn't. For example when A is empty.
    $endgroup$
    – William Elliot
    Jan 13 at 3:18










  • $begingroup$
    You dont need the contradiction. Once you show that $N_r(a)$ exists and misses A, you've shown that X$^c$ is open. Thus X is closed.
    $endgroup$
    – Joel Pereira
    Jan 13 at 3:19










  • $begingroup$
    @WilliamElliot but if A is empty then X is closed.
    $endgroup$
    – mathsssislife
    Jan 13 at 3:21










  • $begingroup$
    @WilliamElliot what do you suggest I do differently?
    $endgroup$
    – mathsssislife
    Jan 13 at 3:22










  • $begingroup$
    Topologically limit points are nearly useless. It is weird that limit points play such a central role in metric space theory. They are clumbsy and their use is not helpful as an introduction to topology. A better alternative for analysis than limit points is punctured nhoods, to be used only as needed for those aspects of analysis that topology is less concerned about. @mathsssislife
    $endgroup$
    – William Elliot
    Jan 13 at 9:11


















0












$begingroup$


I have a question with regards to the proof of the following:
Let $X = A cup A'$ be the closure of a set. $A'$ is the set of limit points of $A$. I wish to show that the closure is closed. I would very much like your opinions on the proof.



Let $ain A^ccap (A')^c$. Since $a$ is not a limit point of $A$ there exists a neighborhood of $a$ such that for every $q$ in the neighborhood of $a$, $q=a$ or $qnotin A$. This implies that $N_r(a)$ is contained within the complement of $A$. Now we would like to show that the neighborhood is contained within $(A')^c$, because then $N_r(a)$ $subset$ $A^c$ $cap$ $(A')^c$ which implies that the set is open, therefore the closure is closed. So assume that the neighborhood is not contained in $(A')^c$ this implies that $a$ $in$ $A'$ because $N_r(a)$ $subset$ $A'$, which is a contradiction.










share|cite|improve this question











$endgroup$












  • $begingroup$
    You start by basically assuming a not in X and proceed to make contradicition; thusly proclaiming X is the whole space, which usually it isn't. For example when A is empty.
    $endgroup$
    – William Elliot
    Jan 13 at 3:18










  • $begingroup$
    You dont need the contradiction. Once you show that $N_r(a)$ exists and misses A, you've shown that X$^c$ is open. Thus X is closed.
    $endgroup$
    – Joel Pereira
    Jan 13 at 3:19










  • $begingroup$
    @WilliamElliot but if A is empty then X is closed.
    $endgroup$
    – mathsssislife
    Jan 13 at 3:21










  • $begingroup$
    @WilliamElliot what do you suggest I do differently?
    $endgroup$
    – mathsssislife
    Jan 13 at 3:22










  • $begingroup$
    Topologically limit points are nearly useless. It is weird that limit points play such a central role in metric space theory. They are clumbsy and their use is not helpful as an introduction to topology. A better alternative for analysis than limit points is punctured nhoods, to be used only as needed for those aspects of analysis that topology is less concerned about. @mathsssislife
    $endgroup$
    – William Elliot
    Jan 13 at 9:11
















0












0








0





$begingroup$


I have a question with regards to the proof of the following:
Let $X = A cup A'$ be the closure of a set. $A'$ is the set of limit points of $A$. I wish to show that the closure is closed. I would very much like your opinions on the proof.



Let $ain A^ccap (A')^c$. Since $a$ is not a limit point of $A$ there exists a neighborhood of $a$ such that for every $q$ in the neighborhood of $a$, $q=a$ or $qnotin A$. This implies that $N_r(a)$ is contained within the complement of $A$. Now we would like to show that the neighborhood is contained within $(A')^c$, because then $N_r(a)$ $subset$ $A^c$ $cap$ $(A')^c$ which implies that the set is open, therefore the closure is closed. So assume that the neighborhood is not contained in $(A')^c$ this implies that $a$ $in$ $A'$ because $N_r(a)$ $subset$ $A'$, which is a contradiction.










share|cite|improve this question











$endgroup$




I have a question with regards to the proof of the following:
Let $X = A cup A'$ be the closure of a set. $A'$ is the set of limit points of $A$. I wish to show that the closure is closed. I would very much like your opinions on the proof.



Let $ain A^ccap (A')^c$. Since $a$ is not a limit point of $A$ there exists a neighborhood of $a$ such that for every $q$ in the neighborhood of $a$, $q=a$ or $qnotin A$. This implies that $N_r(a)$ is contained within the complement of $A$. Now we would like to show that the neighborhood is contained within $(A')^c$, because then $N_r(a)$ $subset$ $A^c$ $cap$ $(A')^c$ which implies that the set is open, therefore the closure is closed. So assume that the neighborhood is not contained in $(A')^c$ this implies that $a$ $in$ $A'$ because $N_r(a)$ $subset$ $A'$, which is a contradiction.







real-analysis general-topology analysis proof-verification proof-writing






share|cite|improve this question















share|cite|improve this question













share|cite|improve this question




share|cite|improve this question








edited Jan 13 at 2:59









Janitha357

1,756516




1,756516










asked Jan 13 at 2:39









mathsssislifemathsssislife

437




437












  • $begingroup$
    You start by basically assuming a not in X and proceed to make contradicition; thusly proclaiming X is the whole space, which usually it isn't. For example when A is empty.
    $endgroup$
    – William Elliot
    Jan 13 at 3:18










  • $begingroup$
    You dont need the contradiction. Once you show that $N_r(a)$ exists and misses A, you've shown that X$^c$ is open. Thus X is closed.
    $endgroup$
    – Joel Pereira
    Jan 13 at 3:19










  • $begingroup$
    @WilliamElliot but if A is empty then X is closed.
    $endgroup$
    – mathsssislife
    Jan 13 at 3:21










  • $begingroup$
    @WilliamElliot what do you suggest I do differently?
    $endgroup$
    – mathsssislife
    Jan 13 at 3:22










  • $begingroup$
    Topologically limit points are nearly useless. It is weird that limit points play such a central role in metric space theory. They are clumbsy and their use is not helpful as an introduction to topology. A better alternative for analysis than limit points is punctured nhoods, to be used only as needed for those aspects of analysis that topology is less concerned about. @mathsssislife
    $endgroup$
    – William Elliot
    Jan 13 at 9:11




















  • $begingroup$
    You start by basically assuming a not in X and proceed to make contradicition; thusly proclaiming X is the whole space, which usually it isn't. For example when A is empty.
    $endgroup$
    – William Elliot
    Jan 13 at 3:18










  • $begingroup$
    You dont need the contradiction. Once you show that $N_r(a)$ exists and misses A, you've shown that X$^c$ is open. Thus X is closed.
    $endgroup$
    – Joel Pereira
    Jan 13 at 3:19










  • $begingroup$
    @WilliamElliot but if A is empty then X is closed.
    $endgroup$
    – mathsssislife
    Jan 13 at 3:21










  • $begingroup$
    @WilliamElliot what do you suggest I do differently?
    $endgroup$
    – mathsssislife
    Jan 13 at 3:22










  • $begingroup$
    Topologically limit points are nearly useless. It is weird that limit points play such a central role in metric space theory. They are clumbsy and their use is not helpful as an introduction to topology. A better alternative for analysis than limit points is punctured nhoods, to be used only as needed for those aspects of analysis that topology is less concerned about. @mathsssislife
    $endgroup$
    – William Elliot
    Jan 13 at 9:11


















$begingroup$
You start by basically assuming a not in X and proceed to make contradicition; thusly proclaiming X is the whole space, which usually it isn't. For example when A is empty.
$endgroup$
– William Elliot
Jan 13 at 3:18




$begingroup$
You start by basically assuming a not in X and proceed to make contradicition; thusly proclaiming X is the whole space, which usually it isn't. For example when A is empty.
$endgroup$
– William Elliot
Jan 13 at 3:18












$begingroup$
You dont need the contradiction. Once you show that $N_r(a)$ exists and misses A, you've shown that X$^c$ is open. Thus X is closed.
$endgroup$
– Joel Pereira
Jan 13 at 3:19




$begingroup$
You dont need the contradiction. Once you show that $N_r(a)$ exists and misses A, you've shown that X$^c$ is open. Thus X is closed.
$endgroup$
– Joel Pereira
Jan 13 at 3:19












$begingroup$
@WilliamElliot but if A is empty then X is closed.
$endgroup$
– mathsssislife
Jan 13 at 3:21




$begingroup$
@WilliamElliot but if A is empty then X is closed.
$endgroup$
– mathsssislife
Jan 13 at 3:21












$begingroup$
@WilliamElliot what do you suggest I do differently?
$endgroup$
– mathsssislife
Jan 13 at 3:22




$begingroup$
@WilliamElliot what do you suggest I do differently?
$endgroup$
– mathsssislife
Jan 13 at 3:22












$begingroup$
Topologically limit points are nearly useless. It is weird that limit points play such a central role in metric space theory. They are clumbsy and their use is not helpful as an introduction to topology. A better alternative for analysis than limit points is punctured nhoods, to be used only as needed for those aspects of analysis that topology is less concerned about. @mathsssislife
$endgroup$
– William Elliot
Jan 13 at 9:11






$begingroup$
Topologically limit points are nearly useless. It is weird that limit points play such a central role in metric space theory. They are clumbsy and their use is not helpful as an introduction to topology. A better alternative for analysis than limit points is punctured nhoods, to be used only as needed for those aspects of analysis that topology is less concerned about. @mathsssislife
$endgroup$
– William Elliot
Jan 13 at 9:11












2 Answers
2






active

oldest

votes


















0












$begingroup$

The idea in the first half of the proof showing $N_r(a)subseteq A^c$ is correct. But in my opinion when showing that $N_r(a)$ is not contained in $(A')^c$ your reasoning lacks explanations. I would do it as follows.



Assume $N_r(a)nsubseteq (A')^c.$ Then $N_r(a)cap A'neqemptyset.$ So let $bin N_r(a)cap A'.$ Then there exists $t>0$ such that $N_t(a)subseteq N_r(a)$ (why?). Therefore $N_t(b)cap Aneq emptyset$ (why?). So $N_r(a)cap Aneqemptyset,$ which is a contradiction. Hence $N_r(a)subseteq (A')^c.$






share|cite|improve this answer









$endgroup$













  • $begingroup$
    Would this be better: Let p$in$ $(E cup E′)^c$ which implies that p $notin$ E and p $notin$ E'. Since p$notin$E', $exists$ $N_r(p)$ : $forall$ q$in$ Nr(p) , q $=$ p or q$notin$ E which is equivalent to Nr(p) $ cap$ E $=$ $emptyset$ , therefore Nr(p) ⊂ $E^c$ . Now we would like to show that $N_r(p)$ $subset$ $E′^c$, so assume that is not the case this implies that $N_r(p)$ $subset$ E' $implies$ p $in$ E' which is a contradiction, therefore $N_r(p)$ $subset$ $E^c$ $cup$ E' which means that it is open, therefore the closure is closed.
    $endgroup$
    – mathsssislife
    Jan 13 at 23:46



















0












$begingroup$

So I assume that your definition of "$A$ is closed" is "$A$ contains all its limit points", or equivalently $A' subseteq A$.



If we then define $overline{A}=A cup A'$ then indeed this set is closed:



$$overline{A}' = (A cup A')' = A' cup A'' subseteq A' subseteq overline{A}$$
using that in a $T_1$ space (thus certainly in a metric space) we always have $B'' subseteq B'$ for all subsets $B$ and also $(C cup D)' = C' cup D'$ for all subsets $C,D$. Thus $B = overline{A}$ obeys $B' subseteq B$ so is closed.



Moreover, if $C$ is closed and $A subseteq C $ then $A' subseteq C' subseteq C$, so $overline{A}=A cup A' subseteq C$, so it follows that $overline{A}$ is the smallest closed set that contains $A$ as a subset, another way the closure could have been defined.






share|cite|improve this answer









$endgroup$













    Your Answer





    StackExchange.ifUsing("editor", function () {
    return StackExchange.using("mathjaxEditing", function () {
    StackExchange.MarkdownEditor.creationCallbacks.add(function (editor, postfix) {
    StackExchange.mathjaxEditing.prepareWmdForMathJax(editor, postfix, [["$", "$"], ["\\(","\\)"]]);
    });
    });
    }, "mathjax-editing");

    StackExchange.ready(function() {
    var channelOptions = {
    tags: "".split(" "),
    id: "69"
    };
    initTagRenderer("".split(" "), "".split(" "), channelOptions);

    StackExchange.using("externalEditor", function() {
    // Have to fire editor after snippets, if snippets enabled
    if (StackExchange.settings.snippets.snippetsEnabled) {
    StackExchange.using("snippets", function() {
    createEditor();
    });
    }
    else {
    createEditor();
    }
    });

    function createEditor() {
    StackExchange.prepareEditor({
    heartbeatType: 'answer',
    autoActivateHeartbeat: false,
    convertImagesToLinks: true,
    noModals: true,
    showLowRepImageUploadWarning: true,
    reputationToPostImages: 10,
    bindNavPrevention: true,
    postfix: "",
    imageUploader: {
    brandingHtml: "Powered by u003ca class="icon-imgur-white" href="https://imgur.com/"u003eu003c/au003e",
    contentPolicyHtml: "User contributions licensed under u003ca href="https://creativecommons.org/licenses/by-sa/3.0/"u003ecc by-sa 3.0 with attribution requiredu003c/au003e u003ca href="https://stackoverflow.com/legal/content-policy"u003e(content policy)u003c/au003e",
    allowUrls: true
    },
    noCode: true, onDemand: true,
    discardSelector: ".discard-answer"
    ,immediatelyShowMarkdownHelp:true
    });


    }
    });














    draft saved

    draft discarded


















    StackExchange.ready(
    function () {
    StackExchange.openid.initPostLogin('.new-post-login', 'https%3a%2f%2fmath.stackexchange.com%2fquestions%2f3071639%2fclosure-of-a-set-in-a-metric-space%23new-answer', 'question_page');
    }
    );

    Post as a guest















    Required, but never shown

























    2 Answers
    2






    active

    oldest

    votes








    2 Answers
    2






    active

    oldest

    votes









    active

    oldest

    votes






    active

    oldest

    votes









    0












    $begingroup$

    The idea in the first half of the proof showing $N_r(a)subseteq A^c$ is correct. But in my opinion when showing that $N_r(a)$ is not contained in $(A')^c$ your reasoning lacks explanations. I would do it as follows.



    Assume $N_r(a)nsubseteq (A')^c.$ Then $N_r(a)cap A'neqemptyset.$ So let $bin N_r(a)cap A'.$ Then there exists $t>0$ such that $N_t(a)subseteq N_r(a)$ (why?). Therefore $N_t(b)cap Aneq emptyset$ (why?). So $N_r(a)cap Aneqemptyset,$ which is a contradiction. Hence $N_r(a)subseteq (A')^c.$






    share|cite|improve this answer









    $endgroup$













    • $begingroup$
      Would this be better: Let p$in$ $(E cup E′)^c$ which implies that p $notin$ E and p $notin$ E'. Since p$notin$E', $exists$ $N_r(p)$ : $forall$ q$in$ Nr(p) , q $=$ p or q$notin$ E which is equivalent to Nr(p) $ cap$ E $=$ $emptyset$ , therefore Nr(p) ⊂ $E^c$ . Now we would like to show that $N_r(p)$ $subset$ $E′^c$, so assume that is not the case this implies that $N_r(p)$ $subset$ E' $implies$ p $in$ E' which is a contradiction, therefore $N_r(p)$ $subset$ $E^c$ $cup$ E' which means that it is open, therefore the closure is closed.
      $endgroup$
      – mathsssislife
      Jan 13 at 23:46
















    0












    $begingroup$

    The idea in the first half of the proof showing $N_r(a)subseteq A^c$ is correct. But in my opinion when showing that $N_r(a)$ is not contained in $(A')^c$ your reasoning lacks explanations. I would do it as follows.



    Assume $N_r(a)nsubseteq (A')^c.$ Then $N_r(a)cap A'neqemptyset.$ So let $bin N_r(a)cap A'.$ Then there exists $t>0$ such that $N_t(a)subseteq N_r(a)$ (why?). Therefore $N_t(b)cap Aneq emptyset$ (why?). So $N_r(a)cap Aneqemptyset,$ which is a contradiction. Hence $N_r(a)subseteq (A')^c.$






    share|cite|improve this answer









    $endgroup$













    • $begingroup$
      Would this be better: Let p$in$ $(E cup E′)^c$ which implies that p $notin$ E and p $notin$ E'. Since p$notin$E', $exists$ $N_r(p)$ : $forall$ q$in$ Nr(p) , q $=$ p or q$notin$ E which is equivalent to Nr(p) $ cap$ E $=$ $emptyset$ , therefore Nr(p) ⊂ $E^c$ . Now we would like to show that $N_r(p)$ $subset$ $E′^c$, so assume that is not the case this implies that $N_r(p)$ $subset$ E' $implies$ p $in$ E' which is a contradiction, therefore $N_r(p)$ $subset$ $E^c$ $cup$ E' which means that it is open, therefore the closure is closed.
      $endgroup$
      – mathsssislife
      Jan 13 at 23:46














    0












    0








    0





    $begingroup$

    The idea in the first half of the proof showing $N_r(a)subseteq A^c$ is correct. But in my opinion when showing that $N_r(a)$ is not contained in $(A')^c$ your reasoning lacks explanations. I would do it as follows.



    Assume $N_r(a)nsubseteq (A')^c.$ Then $N_r(a)cap A'neqemptyset.$ So let $bin N_r(a)cap A'.$ Then there exists $t>0$ such that $N_t(a)subseteq N_r(a)$ (why?). Therefore $N_t(b)cap Aneq emptyset$ (why?). So $N_r(a)cap Aneqemptyset,$ which is a contradiction. Hence $N_r(a)subseteq (A')^c.$






    share|cite|improve this answer









    $endgroup$



    The idea in the first half of the proof showing $N_r(a)subseteq A^c$ is correct. But in my opinion when showing that $N_r(a)$ is not contained in $(A')^c$ your reasoning lacks explanations. I would do it as follows.



    Assume $N_r(a)nsubseteq (A')^c.$ Then $N_r(a)cap A'neqemptyset.$ So let $bin N_r(a)cap A'.$ Then there exists $t>0$ such that $N_t(a)subseteq N_r(a)$ (why?). Therefore $N_t(b)cap Aneq emptyset$ (why?). So $N_r(a)cap Aneqemptyset,$ which is a contradiction. Hence $N_r(a)subseteq (A')^c.$







    share|cite|improve this answer












    share|cite|improve this answer



    share|cite|improve this answer










    answered Jan 13 at 3:46









    Janitha357Janitha357

    1,756516




    1,756516












    • $begingroup$
      Would this be better: Let p$in$ $(E cup E′)^c$ which implies that p $notin$ E and p $notin$ E'. Since p$notin$E', $exists$ $N_r(p)$ : $forall$ q$in$ Nr(p) , q $=$ p or q$notin$ E which is equivalent to Nr(p) $ cap$ E $=$ $emptyset$ , therefore Nr(p) ⊂ $E^c$ . Now we would like to show that $N_r(p)$ $subset$ $E′^c$, so assume that is not the case this implies that $N_r(p)$ $subset$ E' $implies$ p $in$ E' which is a contradiction, therefore $N_r(p)$ $subset$ $E^c$ $cup$ E' which means that it is open, therefore the closure is closed.
      $endgroup$
      – mathsssislife
      Jan 13 at 23:46


















    • $begingroup$
      Would this be better: Let p$in$ $(E cup E′)^c$ which implies that p $notin$ E and p $notin$ E'. Since p$notin$E', $exists$ $N_r(p)$ : $forall$ q$in$ Nr(p) , q $=$ p or q$notin$ E which is equivalent to Nr(p) $ cap$ E $=$ $emptyset$ , therefore Nr(p) ⊂ $E^c$ . Now we would like to show that $N_r(p)$ $subset$ $E′^c$, so assume that is not the case this implies that $N_r(p)$ $subset$ E' $implies$ p $in$ E' which is a contradiction, therefore $N_r(p)$ $subset$ $E^c$ $cup$ E' which means that it is open, therefore the closure is closed.
      $endgroup$
      – mathsssislife
      Jan 13 at 23:46
















    $begingroup$
    Would this be better: Let p$in$ $(E cup E′)^c$ which implies that p $notin$ E and p $notin$ E'. Since p$notin$E', $exists$ $N_r(p)$ : $forall$ q$in$ Nr(p) , q $=$ p or q$notin$ E which is equivalent to Nr(p) $ cap$ E $=$ $emptyset$ , therefore Nr(p) ⊂ $E^c$ . Now we would like to show that $N_r(p)$ $subset$ $E′^c$, so assume that is not the case this implies that $N_r(p)$ $subset$ E' $implies$ p $in$ E' which is a contradiction, therefore $N_r(p)$ $subset$ $E^c$ $cup$ E' which means that it is open, therefore the closure is closed.
    $endgroup$
    – mathsssislife
    Jan 13 at 23:46




    $begingroup$
    Would this be better: Let p$in$ $(E cup E′)^c$ which implies that p $notin$ E and p $notin$ E'. Since p$notin$E', $exists$ $N_r(p)$ : $forall$ q$in$ Nr(p) , q $=$ p or q$notin$ E which is equivalent to Nr(p) $ cap$ E $=$ $emptyset$ , therefore Nr(p) ⊂ $E^c$ . Now we would like to show that $N_r(p)$ $subset$ $E′^c$, so assume that is not the case this implies that $N_r(p)$ $subset$ E' $implies$ p $in$ E' which is a contradiction, therefore $N_r(p)$ $subset$ $E^c$ $cup$ E' which means that it is open, therefore the closure is closed.
    $endgroup$
    – mathsssislife
    Jan 13 at 23:46











    0












    $begingroup$

    So I assume that your definition of "$A$ is closed" is "$A$ contains all its limit points", or equivalently $A' subseteq A$.



    If we then define $overline{A}=A cup A'$ then indeed this set is closed:



    $$overline{A}' = (A cup A')' = A' cup A'' subseteq A' subseteq overline{A}$$
    using that in a $T_1$ space (thus certainly in a metric space) we always have $B'' subseteq B'$ for all subsets $B$ and also $(C cup D)' = C' cup D'$ for all subsets $C,D$. Thus $B = overline{A}$ obeys $B' subseteq B$ so is closed.



    Moreover, if $C$ is closed and $A subseteq C $ then $A' subseteq C' subseteq C$, so $overline{A}=A cup A' subseteq C$, so it follows that $overline{A}$ is the smallest closed set that contains $A$ as a subset, another way the closure could have been defined.






    share|cite|improve this answer









    $endgroup$


















      0












      $begingroup$

      So I assume that your definition of "$A$ is closed" is "$A$ contains all its limit points", or equivalently $A' subseteq A$.



      If we then define $overline{A}=A cup A'$ then indeed this set is closed:



      $$overline{A}' = (A cup A')' = A' cup A'' subseteq A' subseteq overline{A}$$
      using that in a $T_1$ space (thus certainly in a metric space) we always have $B'' subseteq B'$ for all subsets $B$ and also $(C cup D)' = C' cup D'$ for all subsets $C,D$. Thus $B = overline{A}$ obeys $B' subseteq B$ so is closed.



      Moreover, if $C$ is closed and $A subseteq C $ then $A' subseteq C' subseteq C$, so $overline{A}=A cup A' subseteq C$, so it follows that $overline{A}$ is the smallest closed set that contains $A$ as a subset, another way the closure could have been defined.






      share|cite|improve this answer









      $endgroup$
















        0












        0








        0





        $begingroup$

        So I assume that your definition of "$A$ is closed" is "$A$ contains all its limit points", or equivalently $A' subseteq A$.



        If we then define $overline{A}=A cup A'$ then indeed this set is closed:



        $$overline{A}' = (A cup A')' = A' cup A'' subseteq A' subseteq overline{A}$$
        using that in a $T_1$ space (thus certainly in a metric space) we always have $B'' subseteq B'$ for all subsets $B$ and also $(C cup D)' = C' cup D'$ for all subsets $C,D$. Thus $B = overline{A}$ obeys $B' subseteq B$ so is closed.



        Moreover, if $C$ is closed and $A subseteq C $ then $A' subseteq C' subseteq C$, so $overline{A}=A cup A' subseteq C$, so it follows that $overline{A}$ is the smallest closed set that contains $A$ as a subset, another way the closure could have been defined.






        share|cite|improve this answer









        $endgroup$



        So I assume that your definition of "$A$ is closed" is "$A$ contains all its limit points", or equivalently $A' subseteq A$.



        If we then define $overline{A}=A cup A'$ then indeed this set is closed:



        $$overline{A}' = (A cup A')' = A' cup A'' subseteq A' subseteq overline{A}$$
        using that in a $T_1$ space (thus certainly in a metric space) we always have $B'' subseteq B'$ for all subsets $B$ and also $(C cup D)' = C' cup D'$ for all subsets $C,D$. Thus $B = overline{A}$ obeys $B' subseteq B$ so is closed.



        Moreover, if $C$ is closed and $A subseteq C $ then $A' subseteq C' subseteq C$, so $overline{A}=A cup A' subseteq C$, so it follows that $overline{A}$ is the smallest closed set that contains $A$ as a subset, another way the closure could have been defined.







        share|cite|improve this answer












        share|cite|improve this answer



        share|cite|improve this answer










        answered Jan 13 at 15:45









        Henno BrandsmaHenno Brandsma

        107k347114




        107k347114






























            draft saved

            draft discarded




















































            Thanks for contributing an answer to Mathematics Stack Exchange!


            • Please be sure to answer the question. Provide details and share your research!

            But avoid



            • Asking for help, clarification, or responding to other answers.

            • Making statements based on opinion; back them up with references or personal experience.


            Use MathJax to format equations. MathJax reference.


            To learn more, see our tips on writing great answers.




            draft saved


            draft discarded














            StackExchange.ready(
            function () {
            StackExchange.openid.initPostLogin('.new-post-login', 'https%3a%2f%2fmath.stackexchange.com%2fquestions%2f3071639%2fclosure-of-a-set-in-a-metric-space%23new-answer', 'question_page');
            }
            );

            Post as a guest















            Required, but never shown





















































            Required, but never shown














            Required, but never shown












            Required, but never shown







            Required, but never shown

































            Required, but never shown














            Required, but never shown












            Required, but never shown







            Required, but never shown







            Popular posts from this blog

            Mario Kart Wii

            What does “Dominus providebit” mean?

            Antonio Litta Visconti Arese